13
$\begingroup$

I don't understand how to calculate this generalized two-point function or propagator, used in some advanced topics in quantum field theory, a normal ordered product (denoted between $::$) is subtracted from the usual time ordered product (denoted $T$):

$$\langle X^{\mu}(\sigma,\tau)X^{\nu}(\sigma',\tau')\rangle ~=~ T ( X^{\mu}(\sigma,\tau)X^{\nu}(\sigma',\tau')) ~-~ : X^{\mu}(\sigma,\tau)X^{\nu}(\sigma',\tau'):$$

My question is can the rhs of this propagator be derived or the meaning of the subtraction of the time ordered product explained and motivated in simple words?

$\endgroup$
0

1 Answer 1

24
$\begingroup$

If the operators $X_i$ can be written as a sum of an annihilation and a creation part$^1$

$$X_i~=~A_i + A^{\dagger}_i, \qquad i~\in ~I, \tag{1}$$

$$\begin{align} A_i|\Omega\rangle~=~0, \qquad & \langle \Omega |A^{\dagger}_i~=~0, \cr \qquad i~\in~&I,\end{align}\tag{2}$$

where

$$\begin{align} [A_i(t),A_j(t^{\prime})] ~=~& 0, \cr [A^{\dagger}_i(t),A^{\dagger}_j(t^{\prime})] ~=~& 0, \cr i,j~\in ~&I,\end{align}\tag{3} $$

and

$$\begin{align} [A_i(t),A_j^\dagger(t^{\prime})] ~=~& (c~{\rm number}) \times {\bf 1},\cr i,j~\in~&I,\end{align}\tag{4} $$

i.e. proportional to the identity operator ${\bf 1}$, then one may prove that

$$\begin{align} T(&X_i(t)X_j(t^{\prime})) ~-~:X_i(t)X_j(t^{\prime}):\cr ~=~&\langle \Omega | T(X_i(t)X_j(t^{\prime}))|\Omega\rangle ~{\bf 1}.\end{align} \tag{5}$$

Proof of eq. (5): On one hand, the time ordering $T$ is defined as

$$\begin{align} T(&X_i(t)X_j(t^{\prime}))\cr ~=~& \Theta(t-t^{\prime}) X_i(t)X_j(t^{\prime}) +\Theta(t^{\prime}-t) X_j(t^{\prime})X_i(t)\cr ~=~&X_i(t)X_j(t^{\prime}) -\Theta(t^{\prime}-t) [X_i(t),X_j(t^{\prime})]\cr ~\stackrel{(1)+(3)}{=}&~X_i(t)X_j(t^{\prime})\cr &-\Theta(t^{\prime}-t) \left([A_i(t),A^{\dagger}_j(t^{\prime})]+[A^{\dagger}_i(t),A_j(t^{\prime})]\right).\end{align} \tag{6}$$

On the other hand, the normal ordering $::$ moves by definition the creation part to the left of the annihilation part, so that

$$\begin{align}:X_i(t)X_j(t^\prime):~\stackrel{(1)}{=}~& X_i(t)X_j(t^{\prime}) \cr ~-~& [A_i(t),A^{\dagger}_j(t^{\prime})], \end{align}\tag{7}$$

$$ \langle \Omega | :X_i(t)X_j(t^{\prime}):|\Omega\rangle~\stackrel{(1)+(2)}{=}~0.\tag{8}$$

The difference of eqs. (6) and (7) is the lhs. of eq. (5):

$$\begin{align} T(& X_i(t)X_j(t^{\prime})) ~-~:X_i(t)X_j(t^{\prime}): \cr ~\stackrel{(6)+(7)}{=}& \Theta(t-t^{\prime})[A_i(t),A^{\dagger}_j(t^{\prime})] \cr ~+~& \Theta(t^{\prime}-t)[A_j(t^{\prime}),A^{\dagger}_i(t)],\end{align}\tag{9}$$

which is proportional to the identity operator ${\bf 1}$ by assumption (4). Now sandwich eq. (9) between the bra $\langle \Omega |$ and the ket $|\Omega\rangle $. Since the rhs. of eq. (9) is proportional to the identity operator ${\bf 1}$, the unsandwiched rhs. must be equal to the sandwiched rhs. times the identity operator ${\bf 1}$. Hence also the unsandwiched lhs. of eq. (9) must also be equal to the sandwiched lhs. times the identity operator ${\bf 1}$. This yields eq. (5). $\Box$

A similar argument applied to eq. (7) yields that

$$\begin{align} X_i(t)&X_j(t^{\prime}) ~-~:X_i(t)X_j(t^{\prime}):\cr ~=~&\langle \Omega | X_i(t)X_j(t^{\prime})|\Omega\rangle ~{\bf 1} \end{align}\tag{10}$$

i.e. a version of eq. (5) without the time-order $T$.

--

$^1$ The operators $A_i$ and $A^{\dagger}_i$ need not be Hermitian conjugates in what follows. We implicitly assume that the vacuum $|\Omega\rangle$ is normalized: $\langle \Omega | \Omega\rangle=1$.

$\endgroup$
3
  • 1
    $\begingroup$ Thanks a lot @Qmechanic, this clear proof is exactly what I needed. The only thing left to do for me now is to check that the preconditions are valid in my specific case. $\endgroup$
    – Dilaton
    Dec 10, 2011 at 10:40
  • $\begingroup$ @Qmechanic, for the operators $A$ and $B$, is the equation $\langle 0|A+B|0\rangle=\langle 0|A|0\rangle+\langle 0|B|0\rangle$ satisfied? If it is satisfied, then we can sandwich the lhs. of eq. (1) and the equation simply becomes $\langle 0|:X_i(t)X_j(t'):|0\rangle=0$. But such equation sometimes fails. $\endgroup$
    – Wein Eld
    Dec 28, 2015 at 11:30
  • $\begingroup$ I updated the answer. It seems you are talking about situations where one uses definitions of Fock vacuum and normal order that are not properly compatible/adjusted to each other. $\endgroup$
    – Qmechanic
    Dec 28, 2015 at 14:17

Your Answer

By clicking “Post Your Answer”, you agree to our terms of service and acknowledge you have read our privacy policy.

Not the answer you're looking for? Browse other questions tagged or ask your own question.